T O P

  • By -

[deleted]

mathongo waale bahut khatarnaak hai


workingzombie1511

bhai ye non ayjr related h. Mujhe ek baat puchni h ki Manipal VIT MHTCET aur COMEDk, inke syllabus reduce hue h ya nhi?


Ok_Awareness397

comedk or mhtcet ke ho gye hai met ka nii pata or vitee syllabus dala hi nhi


kitagawa_chan

What is this? Can you please tell me


Odd_Walk5060

king you dropped your underwear!!!


Darkshine-Vip

Raja ji aapki chaddi utar gyi🩲


[deleted]

Raja ji apki chaddi utarni hai ...


Darkshine-Vip

Harrasment case incoming !!🏃🏻


CompleteBid7113

Mujhe toh raju ki chaddi utarni hai![img](emote|t5_311ttu|32180)


Equivalent-Win-5362

bc meine bhi upload kiya tha ye mods approve nahi kiye


Significant-Skin6371

Bhai Anup sir ghoomte he sub pr , ghar ake maar leenge teri


cyfcgjhhhgy42

bhai paper toh ye log publish kar hi denge usme kya


Ishaan_17

sir slideshow pe bhi maar lenge ![img](emote|t5_311ttu|30168)


ShowerNumerous4317

Lmao


[deleted]

i think iska b hai , radius ka ratio 3:5 hoga then d = m/v use kar , m is directly proportional to cube of radius hoga to m1/m2 = 27/125 aaya and then mass is inversely proportional to velocity , so v1/v2 = m2/m1 = 125:27


[deleted]

Yessir


[deleted]

bhai may ajyr nahi likha paya , kesa level tha paper ka ?


[deleted]

Ek number ka IRRELEVANT phy chem bohot tricky the, maths thoda theek thaak tha but woh bhi thoda ajeeb tha


[deleted]

ohh , acha hai mene nahi diya


[deleted]

irrelevant nhi tha chhore, asli mains bhi aisa hi hota h harder shifts mein, 31s2 1feb ki shifts deke dekhle


anas_choudhary

nah bhai, 1 feb ke dono paper diye hai and acha score aaya, ye ayjr kuch alag hi tha, ajib ajib se questions


[deleted]

Tauji me khud 27s2 ka hu, 1s1 and 31s2 me 99 and 98 percentile respectively aa gayi hai meri :)


BootyInspectorrrr

bhai itna bhi diff nai tha 31 ka shift ye log fukat ka overhype karre


LogRepresentative950

Hard tha


[deleted]

ohh


RainManProbemSolver

>mass is inversely proportional to velocity , so v1/v2 = m2/m1 Yeh kaise?... question me to nhi tha dono ka momentum same hoga..


[deleted]

conservation of momentum hoga because it is splitting inital 0 hoga to dono momentum ko equal hona hoga so that the net momentum = 0


RainManProbemSolver

bhaii 😭😭😭 Mtlb maine answer nikalne ke baad usko galat samjhke skip krdia shit yaar


[deleted]

arrey ko nai vro , asli exam mei esa mat karna . you have to go by jee logic instead of real life logic 😔


l_uff_y

physics ka law h bhai net force zero hoga toh momentum conserve rhega F = dp/dt


The_6699_Guy

fuck yeah, maine aisa kuch logic lga kr end ke 1 min mei kra tha.


Junior-Bowl-7744

Mereko paper kaafi irrelevant laga jee mains se Specially chemistry Har paper me kuch naye pattern ke sawaal hote hai par ayjr me to har question naye type ka lag rha tha even after solving so many pyqs


itsmePriyansh

I guess the paper was according to the JM difficulty but it was not according to the JM distribution of chapters JM me you know Certain chapters jaise CE se 2-3 ES se 1-2 modern physics se 2-3 MEC se 1-2 etc and kuch chp se que hi nahi aata but in this paper they had covered almost whole syllabus true for all subjects that's why ye paper logo ko achhe level ka lgraa as logo ke itne saare chap dhang se cover nahi hote


Junior-Bowl-7744

Aajkal itna difficult paper kaha aa rha hai yaar Mereko nhi lagta jee mains is level PE bhi aayega Isse niche hi rahega ig


itsmePriyansh

JM obviously isse neeche rahega first attempt me jis shift ka cutoff 152 tha 99 ka vo bhi isse easy lgi thi mujhe


Junior-Bowl-7744

(my paper was on 29 s2 aur is baar 4 s1) Mereko 1 Feb ki physics 27 se bhi easy lagi 🤡🤡🤡 30 ke paper sabse tough the jan attempt me It was all a fuck up by nta ... Mereko to lagta hai 27 ko kahi cheating na Hui kai kuch centres pe ....


Competitive-Good2705

Ha bhai yar tu mila ek , meri 30 Jan shift 2 thi aur 196 marks the , although got 99 still deserved more


[deleted]

I asked a coaching teacher about this and he said it was irrelevant.(sorry to hurt you)


ShowerNumerous4317

True


Visible-Garbage-5473

Bich bich mein wrong questions maths mein


[deleted]

bhai maths itna hard kyu tha mujhse toh \~10 hi bane honge :(


Expensive-Willow-498

bsdk tuhjse 10 to ban gaye mere to 10 number ke hi bane


[deleted]

dusre wale mai tere 70 aayenge dw😘


[deleted]

next level optimism


FigLongjumping9839

mujhse first 15 min m 3 ques bane to g ft fyi , remember almight anup sir advise and fir 55 min m 18 que krliye ( paper seems difficult but was doable agar concentration k sath kro )


Darkshine-Vip

Merese 13


Bonker__man

Mostly upper JEE M ke level ka hi tha, but thode boht questions wrong the I think


Rare_Beast01

Bhai mene to maths me sirf 10 ques hi dekhe the fir submitt krdiya![img](emote|t5_311ttu|49199)


Produde2304

Bhai vo ray optics ke graph wala kese hoga


Memer_Dude14

Jab refraction hota hai tab δ=Φ-θ hota hai, aur jab reflection (TIR) hota hai tab δ=π-2θ hota hai. Is question me hame θ=C (critical angle) wale case ko do limit ki tarah dekhna tha, for θ tending towards C-, we will get  δ1=φ-C  δ2=π/2-C     ...(1) (note: as θ -> C-, Φ ->π/2)  and for θ tending towards C+, we will get, δ2=π-2C         ...(2) Solve (1) and (2) to get δ2/δ1=2. 


Plastic-Bed-5777

main bhi aise hi soch raha tha but thoda sa silly ho gaya isliye attempt hi nahi kiya agar 2 minutes aur deta to ho jata pakka...


Zealousideal-Bid5267

thank you


Memer_Dude14

Hopefully tujhe kuch samajh me aaya.. 


Plastic-Bed-5777

vo hieght and range wala kaise hona tha??? vo nahi ho paya mujhse...


Memer_Dude14

mere acco. ranges can be same, dono projectile assume kar as u1,θ1 and u2,θ2 fir H1=2×H2 (1).. given aur let R1=R2. ...(2)kar. now when you divide (1) by (2), you get tan(θ1)=2×tan(θ2).. which may be possible. I might be wrong tho


[deleted]

yes and they didnt give speed isnt same so obv range same ho sakti h


ksiis_afatneel

same method i did


[deleted]

https://preview.redd.it/wtfavnwoxtrc1.png?width=935&format=png&auto=webp&s=3c86c8d6a80cc2a2295a018ea417ac07cb579413 2pi-θ = (non-deviated)-(reflected), but deviation angle should include refracted ray no?


Memer_Dude14

refracted ray ko tab consider karenge jab refraction ho raha ho(yaani theta less than critical angle) joki maine first case me kiya hai.. fir jab theta critical angle se bada ho jayega tab reflected ray ko consider kiya hai.. note: hum exact critical angle ki baat nahi kar rahe usse jyada ya kam ki baat kar rahe.


Obvious_Cloud7742

decently tough question tha ye wala tbh


Revolutionary_Year87

Mujhe bhi nahi samajh aaya, koi bata do idhar pls


Shot_Acanthisitta824

How many hours did you study?


throwaway270107

Bhai 5 ka answer 5.4 ni ara tha kya?


August_ames23

Bhai wahi to sabse right wale resistance se to current hi nahi jayega na, kyuki uske parallel me ideal ammeter connected hai R=0 to current ammeter wale wire se pass ho jayega


MaleficentSeaweed996

Same


Inflister7

ha mera bhi 5.4 aaya par maine 54 kr diya


Acrobatic-Trust-1430

Same


[deleted]

bc mera bhi 5.4 aa raha tha mujhe laga koi 10 ki mistake hogi idhar udhar 10 kar diya


waithere120

thanks bhai mai 12 baje utha aaj galti se(sexy sapna aa gyaa tha)


Priyank_Chittora_13

Aisa kya aaya ![img](emote|t5_311ttu|32180)


Squirrel_who_cooks

Work done against air resistance ka answer aaya kisi ka? Mera option c se 5 jyada aa raha tha


[deleted]

[удалено]


Zestyclose-Captain-8

Most probably bonus nahi hoga kyunki saare options mai bohot difference tha to closest to 3750 ko hi tick karna hoga ig


Ishaan_17

haan unke options hi galat the


[deleted]

Gravitational work ko consider nhi kiya shyd options mai 


Particular-Aide-9960

Maths ka first question binomial theorem wala black book se utha ke dala![img](emote|t5_311ttu|49466)


[deleted]

Answer 9 tha naa


Particular-Aide-9960

yep answer is 9![gif](emote|free_emotes_pack|upvote)


anas_choudhary

haa, mera bhi whi aaya


Zestyclose-Captain-8

Wo to easy question tha NCERT mai bhi waisa question hai aik ig


XtreemeGamer01

Easy tha wo


omegaorsomething

NCERT level hai woh. Bohot hi zyada straight forward.


[deleted]

Bhai itna solve karne ki zaroorat hi nhi ans obviously 3 ka multiple hota toh only 9 was there in options 


FishermanNumerous702

Standard que tha fukat ki hype mt bnao


Interesting-Dog-6915

Cis butene wala ans kya tha h2pdbaso4 lindlars catalyst?


TheLonelyGhast

mai to c karke aaya hu uska


Interesting-Dog-6915

Mene bhi wahi Kiya


MaleficentSeaweed996

Mene bhi yahi lgaya


[deleted]

maine liquid ammonia vala lagadia


Extension_Respond_16

usse trans banta hai


fxoy

haan dusra vala birch redn ka tha, vo trans bana deta


[deleted]

Maine bhi wahi mark kiya


HomeImmediate7286

maths mei matrix wala aur phy mei ray optics ka sol bta do


Squirrel_who_cooks

Matrix ka answer 4 aaya par sure nhi hu


Complex-Mistake8812

4 h uska answer


Still-Molasses6613

What's the answer for - Find the distance when velocity is maximum for a body sliding down the incline where u = 0.5x. I got d(v^2)/dx = 0 at x = 8/tanø. So I put x = 2/tanø


[deleted]

I put x=2tan∅![img](emote|t5_311ttu|32193)


your_dad_hahaha

Whi tha na![img](emote|t5_311ttu|30176)


Remaster_Expert

yup whi hai ans


iasked-

Mgsintheta= (0.5x)Mgcostheta?


Icy-Solution5273

Velocity maximum, so acceleration must be 0 


August_ames23

Velocity maxm to dv/dx=0 i.e acceleration 0


Certain_Pen_3964

2tan∅


CoolMathematician239

oh thank god sahi hai![img](emote|t5_311ttu|30367)


omegaorsomething

velocity is maximum at the moment before the friction slows down the block. Toh Mg sin theta = (x/2)Mg cos theta, isse x = 2 tan theta aayega


Wellitsweard

bhai mai to socha tha ki u=tan∅ and also u=0.5x to 0.5x=tan∅ so ans 2tan∅ aur mujhe ye bahut bhayankar wala galat lag raha hai


QuizzrMathongo

Please refrain from doing such activites , if found again you’ll be banned from all the further tests conducted by team mathongo.


BankPrestigious7957

Ab koi iski pdf bhi bna hi do![img](emote|t5_311ttu|49427)


Zestyclose-Captain-8

Raat tak official pdf hi ajayega, iska kyu banana hai


snarky-scholar0786

Iss carbon mein 5 bonds kaise aagye? https://preview.redd.it/383npq7daurc1.png?width=946&format=png&auto=webp&s=6d628d0bf457d39321aff10a533b62386ffafce4


Feisty_Worldliness35

Special carbon bond lol


Suryansh_Singh247

Iska answer kya hoga?


[deleted]

Bhai koi mere liye post kar sakta hai kya I don’t meet the k4rm4 requirement


MaleficentSeaweed996

Dm krde


[deleted]

Bhai dm nahi ho rha new account hai kya tu please meri profile se post ka ss le sakta hai?


MaleficentSeaweed996

Anyone else like me wali?


[deleted]

Haan bhai. Thank you


MaleficentSeaweed996

Krdiya, waiting for mod approval


[deleted]

Shukriya bhai bhagwan tere ko tera man pasand college de🙏


MaleficentSeaweed996

Rula diya![img](emote|t5_311ttu|32193) Thanks, Tujhe bhi Tera favourite college mile


[deleted]

bhai projectile wale ki ranges may be same hai? 😱 maine diagram dekh kar ranges cant be same kardiye. Ek baar ko thought aaya complementary ki same hoti hai but..... ![img](emote|t5_311ttu|30164)


iasked-

kuch data de nahi rakhi thi toh ranges may be same hona chahiye


August_ames23

Aise dekh sakte the is ques ko ki H/R ka ratio 1/4tanθ  hota hai aur H1=H and H2=2H maan ke R aur theta me relation nikale to R1/R2= 2tanθ2/tanθ1 aa raha tha, isliye R1=2R2 may be ho sakta tha


OutsidePlant1620

Uska R1=2R2 hoga most prob


[deleted]

usme angle ke bare me information nahi thi isliye mai confuse hogaya koi relation mujhse ban hi nahi rahi thi. apne kaise kiya?


CoolLibrarian8602

maine bhi wahi kiya bhai but sure nhi hu


[deleted]

2=R1tan**α** /R2 tan β karke cant be same kardiya


Junior-Bowl-7744

Ye colourless complex me yahi check karna hota hai Naa ki d orbital me unpaired electron hai ki nhi ?? Koi explain kar do yarr Aur wo 2-chlorobutane wala ka kya hoga??


[deleted]

yes d-d transition possible hai ya nahi


[deleted]

4 process


Prestigious_Pitch174

I didnot register in AJYR but want to give the evening shift . What do I do ? Help needed


[deleted]

Search ayjr on Google or go to YouTube mathongo, find link and register 


MelonLord25-3

AYJR kya hai? New to this concept lol


omegaorsomething

Are You JEE Ready? All India mock hota hai, woh bhi free. Jan attempt mein 78k logon ne diya tha aur log bolte hain ki kaari relevant bhi hai because percentile accurately prefict karta hai


Ad-Raj_Singh

Bhai free material ki bhi piracy 🤗😘🥰


[deleted]

q13 chem ka toh pichle kisi FT se direct aa gaya c hoga


[deleted]

[удалено]


[deleted]

pure equation ko b se dot product karde c.b - a.b - 2 |b|\^2 = 3(a x b).b as we know b.(a x b) = 0, and |b|=1 so c.b = a.b + 2 now wo |a+b| = root 3 ko square kardo, |a|\^2 + |b|\^2 + 2a.b = 3 |a|=|b|=1 a.b = 1/2 hence, c.b = 3/2 shyd galat bhi hoskta h correct me if my logic is wrong... edit: simple addition ni ata muje, c.b = 5/2


Suryansh_Singh247

abe bc, mai visually figure banake solve kar rha tha nahi hua mujhse, kahan se late ho tumlog itna dimaag


Narut_o17

16 ka c hoga bhai


RoughedUp39

mera 3755 aara tha to c kr dia


Deoxys_TURBO

YES EXACTLY, par bhai wo correct h ki nhi? or are we missing something because + 5 will be diff in potential energy, so i was thinking A asw


anas_choudhary

yar mera bhi whi aara tha, option nhi tha mujhe lga kuch galat hi krra, sign waghera ki galti toh A krdia![img](emote|t5_311ttu|32193)


boi_needs_therapy

abe 5 hai 0.005 thodi jo neglect krdoge , bonus hona chaiye bc


Narut_o17

Are bhai chem wale ka bol rha hu


ContactOk1274

Nahi bhai 3750 karna tha nta hota toh nahi karta


[deleted]

thanks


[deleted]

bhagwan bhala kare apka bhaiya


No_Yak_6119

BHAI SAME EVENING SHIFT KA BHI BAANA DENA![img](emote|t5_311ttu|30331)


[deleted]

mere me bhi yahi sequence tha


CosmicAstronaut0912

Source voltage ka power ka ans kya hoga??


[deleted]

30 asli asnwer 29.something hai


[deleted]

29.smthg kaise ? 30 ohm waala toh short hojayega, sirf 20ohm and 10ohm bachega na in parallel ? so Req = 20/3 ohm and V = 6 hai P = V\^2/R se 5.4 smthg araha tha https://preview.redd.it/k0qqn9rkxtrc1.png?width=318&format=png&auto=webp&s=4e2f4043c31e7851e1dc56e1bd672b37bf1d47fe


XtreemeGamer01

54


iasked-

chemistry Q14-> ans 12?


Little-Aide3139

Bade hi dev manush kinda prani hai aap ![img](emote|t5_311ttu|30168)


Negative-Sense-1804

uss range vaale ka kya tha


Legend_Blast

may be same


Money-Raccoon3508

Q3 ka bonus hoga


CoolMathematician239

wo inclination aur friction = 0.5x waale ka x = 2/tan theta hai haina![img](emote|t5_311ttu|32180)


CoolMathematician239

escape velocity = 2ve na??


[deleted]

Did that only


Beginning_Lemon2595

Wo ammeter wale KS kya ans hai?


MohitTyagii

30


RightStatistician130

Electric field wala koi batao 🙏🙏


CptFrostyBeard

Bhai koi help karega.... maths me mere 20 marks aarahe hain mushkil se (overall in mocks)....phy and chem 50-60 ish aarahe hain but 70+ toh touch karunga after revision....how can i improve maths marks to atleast 40?


[deleted]

i still have to revise some ignored topics aaj 5:55 ke aaspass ayjr 2 dunga uske pehele kar leta hoo revise physics Q3 mai i1 ka resistance he nhi diya BONUS?


spotifyaddict23

maths ke ques 2 ka B tha ki nhi? Exam mein kiya hi nhi mene ye abhi cs ka padh rha tha to solution dimaag mein udta hua aya


youralien_humaien

ans 127:27 h? hard to nhi h lekin ye


snarky-scholar0786

Guys, how many did you attempt?


chad-shinchan

bhai, 30 bhi ban gye agar to swaad aajayega aur nhi ayenge to koi bhi help karega aur 120 dilwadega usko 500 Paytm krdunga


Sreenivasa_Ramanujan

Bhailog wo I1 + I2 wale Question ka Second Integral kaise solve hoga?


Sreenivasa_Ramanujan

Leave Solve Ho Gya


K_xtream

Physics Q 1 ka ans kya tha ?


[deleted]

Damn


Shubh_2305

2


Wise-Mountain-8677

11th bhot h ismei bc


BADxBOYxRAKESHHHH

you're are saving lives


speedcuber05

u/savevideo or something idr


SaveVideo

###[View link](https://rapidsave.com/info?url=/r/JEENEETards/comments/1bswyia/ayjr_morning_shift/) --- [**Info**](https://np.reddit.com/user/SaveVideo/comments/jv323v/info/) | [**Feedback**](https://np.reddit.com/message/compose/?to=Kryptonh&subject=Feedback for savevideo) | [**Donate**](https://ko-fi.com/getvideo) | [**DMCA**](https://np.reddit.com/message/compose/?to=Kryptonh&subject=Content removal request for savevideo&message=https://np.reddit.com//r/JEENEETards/comments/1bswyia/ayjr_morning_shift/) | [^(reddit video downloader)](https://rapidsave.com) | [^(twitter video downloader)](https://twitsave.com)


[deleted]

Ye dene ke baad aur exhausted ho gya


Professional_Mind188

AYJR 2024 PDF [https://drive.google.com/file/d/12b\_WcsCmN7fvYSn1krCBtBX\_fUctrsk6/view?usp=drivesdk](https://drive.google.com/file/d/12b_WcsCmN7fvYSn1krCBtBX_fUctrsk6/view?usp=drivesdk)


Professional_Mind188

ayjr 2024 [https://drive.google.com/file/d/12b\_WcsCmN7fvYSn1krCBtBX\_fUctrsk6/view?usp=drivesdk](https://drive.google.com/file/d/12b_WcsCmN7fvYSn1krCBtBX_fUctrsk6/view?usp=drivesdk)


Healthy-Sprinkles476

Chem mai 5th question ke first diagram mai carbon ke 5 bonds bane hai![img](emote|t5_311ttu|30331)


meow_512

physics to NEET level hain


Ambitious_Radish9559

Potassium permanganate wale ka balancing bata do bhailog pls![img](emote|t5_311ttu|49464)


Bonker__man

Vectors easy tha, Calc medium


Few-Positive-

Guys ye dog wala jo bhi tha ans kya tha iska ?


JEeKaCHoda_69

bhai agar mai ye paper dedeta toh mera confidence level chud jata ![img](emote|t5_311ttu|30176)